Difference between revisions of "2005 AMC 12A Problems/Problem 20"
Armalite46 (talk | contribs) (→Solution) |
(→Solution 2) |
||
(3 intermediate revisions by 2 users not shown) | |||
Line 9: | Line 9: | ||
(\text {A}) \ 0 \qquad (\text {B}) \ 2005 \qquad (\text {C})\ 4010 \qquad (\text {D}) \ 2005^2 \qquad (\text {E})\ 2^{2005} | (\text {A}) \ 0 \qquad (\text {B}) \ 2005 \qquad (\text {C})\ 4010 \qquad (\text {D}) \ 2005^2 \qquad (\text {E})\ 2^{2005} | ||
</cmath> | </cmath> | ||
− | == Solution == | + | == Solution 1 == |
For the two functions <math>f(x)=2x,0\le x\le \frac{1}{2}</math> and <math>f(x)=2-2x,\frac{1}{2}\le x\le 1</math>,as long as <math>f(x)</math> is between <math>0</math> and <math>1</math>, <math>x</math> will be in the right domain, so we don't need to worry about the domain of <math>x</math>. | For the two functions <math>f(x)=2x,0\le x\le \frac{1}{2}</math> and <math>f(x)=2-2x,\frac{1}{2}\le x\le 1</math>,as long as <math>f(x)</math> is between <math>0</math> and <math>1</math>, <math>x</math> will be in the right domain, so we don't need to worry about the domain of <math>x</math>. | ||
Also, every time we change <math>f(x)</math>, the expression for the final answer in terms of <math>x</math> will be in a different form(although they'll all satisfy the final equation), so we get a different starting value of <math>x</math>. Every time we have two choices for <math>f(x</math>) and altogether we have to choose <math>2005</math> times. Thus, <math>2^{2005}\Rightarrow\boxed{E}</math>. | Also, every time we change <math>f(x)</math>, the expression for the final answer in terms of <math>x</math> will be in a different form(although they'll all satisfy the final equation), so we get a different starting value of <math>x</math>. Every time we have two choices for <math>f(x</math>) and altogether we have to choose <math>2005</math> times. Thus, <math>2^{2005}\Rightarrow\boxed{E}</math>. | ||
+ | |||
+ | Note: the values of x that satisfy <math>f^{[n]}(x) = \frac {1}{2}</math> are <math>\frac{1}{2^{n+1}}</math>, <math>\frac{3}{2^{n+1}}</math>, <math>\frac{5}{2^{n+1}}</math>, <math>\cdots</math> ,<math>\frac{2^{n+1}-1}{2^{n+1}}</math>. | ||
+ | |||
+ | == Solution 2== | ||
+ | |||
+ | |||
+ | We are given that <math>f^{[2005]}(x) = \frac {1}{2}</math>. Thus, <math>f(f^{[2004]}(x))=\frac{1}{2}</math>. Let <math>f^{[2004]}(x)</math> be equal to <math>y</math>. Thus <math>f(y)=\frac{1}{2}</math> or <math>y=\frac{1}{4}</math> or <math>\frac{3}{4}</math>. Now we know <math>f^{[2004]}(x)</math> is equal to <math>\frac{1}{4}</math> or <math>\frac{3}{4}</math>. Now we know that <math>f(f^{[2003]}(x))=\frac{1}{4}</math> or <math>\frac{3}{4}</math>. Now we solve for <math>f^{[2003]}(x)</math> and let <math>f^{[2003]}(x)=z</math>. Thus <math>f(z)</math> is equal to <math>\frac{1}{8}</math>,<math>\frac{7}{8}</math>,<math>\frac{5}{8}</math>,and <math>\frac{3}{8}</math>. As we see, <math>f^{[2005]}(x)</math> has 1 solution, <math>f^{[2004]}(x)</math> has 2 solutions, and <math>f^{[2003]}(x)</math> has 4 solutions. Thus for each iteration we double the number of possible solutions. There are 2005 iterations and thus the number of solutions is <math>2^{2005}</math> <math>\Rightarrow\boxed{E}</math> | ||
+ | (not rigorous) | ||
==See Also== | ==See Also== |
Latest revision as of 22:49, 28 September 2022
Contents
Problem
For each in , define Let , and for each integer . For how many values of in is ?
Solution 1
For the two functions and ,as long as is between and , will be in the right domain, so we don't need to worry about the domain of .
Also, every time we change , the expression for the final answer in terms of will be in a different form(although they'll all satisfy the final equation), so we get a different starting value of . Every time we have two choices for ) and altogether we have to choose times. Thus, .
Note: the values of x that satisfy are , , , ,.
Solution 2
We are given that . Thus, . Let be equal to . Thus or or . Now we know is equal to or . Now we know that or . Now we solve for and let . Thus is equal to ,,,and . As we see, has 1 solution, has 2 solutions, and has 4 solutions. Thus for each iteration we double the number of possible solutions. There are 2005 iterations and thus the number of solutions is (not rigorous)
See Also
2005 AMC 12A (Problems • Answer Key • Resources) | |
Preceded by Problem 19 |
Followed by Problem 21 |
1 • 2 • 3 • 4 • 5 • 6 • 7 • 8 • 9 • 10 • 11 • 12 • 13 • 14 • 15 • 16 • 17 • 18 • 19 • 20 • 21 • 22 • 23 • 24 • 25 | |
All AMC 12 Problems and Solutions |
The problems on this page are copyrighted by the Mathematical Association of America's American Mathematics Competitions.